LSAT and Law School Admissions Forum

Get expert LSAT preparation and law school admissions advice from PowerScore Test Preparation.

User avatar
 Dave Killoran
PowerScore Staff
  • PowerScore Staff
  • Posts: 5852
  • Joined: Mar 25, 2011
|
#44160
Complete Question Explanation
(The complete setup for this game can be found here: lsat/viewtopic.php?t=13729)

The correct answer choice is (A)

Answer choice (A) is the correct answer.

Answer choice (B) is incorrect because M can only have one connection.

Answer choice (C) is incorrect because H and M cannot be connected, as established in the discussion above about the third rule.

Answer choice (D) is incorrect because P cannot be connected to both T and V.

Answer choice (E) is incorrect because H and T cannot be connected.

Get the most out of your LSAT Prep Plus subscription.

Analyze and track your performance with our Testing and Analytics Package.